University Calculus: Early Transcendentals (3rd Edition)

Published by Pearson
ISBN 10: 0321999584
ISBN 13: 978-0-32199-958-0

Chapter 7 - Section 7.3 - Hyperbolic Functions - Exercises - Page 418: 62

Answer

$$\cosh^{-1}\Big(\frac{5}{3}\Big)=\ln3$$

Work Step by Step

We have $$\cosh^{-1}x=\ln(x+\sqrt{x^2-1})$$ for $x\in[1,\infty)$ Therefore, $$\cosh^{-1}\Big(\frac{5}{3}\Big)=\ln\Big(\frac{5}{3}+\sqrt{\frac{25}{9}-1}\Big)$$ $$=\ln\Big(\frac{5}{3}+\sqrt{\frac{16}{9}}\Big)$$ $$=\ln\Big(\frac{5}{3}+\frac{4}{3}\Big)$$ $$=\ln\frac{9}{3}=\ln3$$
Update this answer!

You can help us out by revising, improving and updating this answer.

Update this answer

After you claim an answer you’ll have 24 hours to send in a draft. An editor will review the submission and either publish your submission or provide feedback.